The smallest number by which 48 should be multiplied so as to get a perfect square is ​

Answers

Answer 1

Answer:

3

Step-by-step explanation:

hrhrhehrhrhrhrhruruurruururuur

The Smallest Number By Which 48 Should Be Multiplied So As To Get A Perfect Square Is

Related Questions

Darcy gave her hairstylist a $ 4.90 The tip was 14​% of the cost of the haircut . Write an equation to find​ b, the cost of the haircut.

Answers

Answer:

Equation: 4.90/b = 14/100

Solution: b = $35

Step-by-step explanation:

Variable b = cost of the haircut

Solve for b:

4.90/b = 14/100

490 = 14b

35 = b

Check your work:

35 × 0.14 = 4.90

Correct!

Simplify the expression.

33 · 32 + 12 ÷ 4

Answers

Answer:

1059

Step-by-step explanation:

33 · 32 + 12 ÷ 4

PEMDAS

Multiply and divide first from left to right

1056 + 3

Then add

1059

[tex]\huge\textsf{Hey there!}[/tex]

[tex]\mathsf{33\times32+12\div4}\\\\\mathsf{33\times32= \boxed{\bf 1,056}}\\\\\mathsf{\bold{1,056}+12\div4}\\\\\mathsf{12\div4=\boxed{\bf 3}}\\\\\mathsf{1,056+\bf 3}\\\mathsf{= \boxed{\bf 1,059}}\\\\\\\boxed{\boxed{\large\textsf{Answer: \huge \bf 1,059}}}\huge\checkmark[/tex]

[tex]\large\textsf{Good luck on your assignment and enjoy your day!}[/tex]

~[tex]\frak{Amphitrite40:)}[/tex]

Stats question what are the main difference between frequenting and Bayesian

Answers

Answer:

The frequentist believes that probability represents long term frequencies of repeatable events (such a flipping a coin). Frequentists do not attach probabilities to hypotheses or unknown values. On the other hand, the Bayesian approach uses probabilities to represent the uncertainty in any event or hypothesis.

HOPE IT HELPS

Can someone answer with steps and explanation? Thanks.

Answers

Answer:

[tex]x=-16\text{ or } x=7[/tex]

Step-by-step explanation:

Since ΔABC is mapped onto ΔDEF, we can write that:

[tex]\Delta ABC\cong \Delta DE F[/tex]

By CPCTC:

[tex]\angle A\cong \angle D[/tex]

And since ΔABC is isosceles with Vertex C:

[tex]\angle A \cong \angle B[/tex]

We are given that:

[tex]m\angle D=34[/tex]

Hence:

[tex]m\angle A=34=m\angle B[/tex]

We are also given that:

[tex]m\angle C=x^2+9x[/tex]

The interior angles of a triangle must sum to 180°. Thus:

[tex]m\angle A+m\angle B+m\angle C=180[/tex]

Substitute:

[tex](34)+(34)+(x^2+9x)=180[/tex]

Simplify:

[tex]68+x^2+9x=180[/tex]

Isolate the equation:

[tex]x^2+9x-112=0[/tex]

Factor:

[tex](x+16)(x-7)=0[/tex]

Zero Product Property:

[tex]x+16=0\text{ or } x-7=0[/tex]

Solve for each case:

[tex]x=-16\text{ or } x=7[/tex]

Testing the solutions, we can see that both yields C = 112°.

Hence, our solutions are:

[tex]x=-16\text{ or } x=7[/tex]

2 divided by 0.75 full divison work i dont just need the answer​

Answers

Answer:

0.375

Step-by-step explanation:

Check the picture below.

whenever we do division of decimals, we have to mind how many decimals are there on each amount, the dividend as well as the divisor, that way we pad with zeros the other amount accordingly whilst losing the dot, for example, to say divide 3 by 0.123, 3 has no decimals, whilst 0.123 has three decimals, so we can just divide 3000 by 0123, so dividing 3 by 0.123 is the same as dividing 3000 by 123.  Another example, if we were to divide say 23.761 by 555.89331, the dividend has 3 decimals, that means 3 zeros the other way, the divisor has 5 decimals, that means 5 zeros the other way while losing the dots, so we'd end up dividing 2376100000 by 55589331000, which we can simplify to just 2376100 by 5589331, as you can see in the picture in this case.

Suppose that g(x)= f(x)+ 6. Which statement best compares the graph of g(x) with the graph of f(x)?

A. The graph of g(x) is the graph of f(x) shifted 6 units down.

B. The graph of g(x) is the graph of f(x) shifted to the right.

C. The graph of g(x) is the graph of f(x) shifted 6 units to the left.

D. The graph of g(x) is the graph of f(x) shifted 6 units up.

Answers

Answer:

D

Step-by-step explanation:

The + 6 moves it up 6 units.

The correct answer is (D) "The graph of g(x) is the graph of f(x) shifted 6 units up."

What is the function?

A relationship between a group of inputs and one output is referred to as a function. In plain English, a function is an association between inputs in which each input is connected to precisely one output. A domain, codomain, or range exists for every function. Typically, f(x), where x is the input, is used to represent a function.

When we add a constant to a function, such as in the case of g(x) = f(x) + 6, it will shift the graph of f(x) upward by 6 units.

This is because, for any value of x, the value of f(x) will be added to 6, resulting in a vertical shift of the entire graph.

Option (A) is incorrect because adding 6 to f(x) would shift the graph up, not down.

Option (B) is incorrect because adding a constant to a function does not cause it to shift horizontally.

Option (C) is incorrect because adding 6 to f(x) would shift the graph right, not left.

D. The graph of g(x) is the graph of f(x) shifted 6 units up. Adding a constant term to a function will shift the graph of the function vertically. In this case, adding 6 to f(x) will shift the graph of f(x) upward by 6 units, resulting in the graph of g(x).

Learn more about function here:

https://brainly.com/question/29633660

#SPJ7

4(x+9)=8x-7
Solve for x

Answers

Answer:

x = 10.75

Step-by-step explanation:

4(x+9)=8x-7

Step 1: Distribute the 4 to the x and 9

4 * x = 4x

4 * 9 = 36

We now have 4x + 36 = 8x - 7

Step 2: add 7 to both sides

36 + 7 = 43

-7 + 7 cancels out

We now have 4x + 43 = 8x

Step 3 subtract 4x from both sides

4x - 4x cancels out

8x - 4x = 4x

We now have 4x = 43

step 4 divide both sides by 4

4x / 4 ( the 4s cancel out and we're left with x )

43/4 = 10.75

We're left with x = 10.75

I need help k please help gardinuhola

Answers

Answer:

Step-by-step explanation:

for circle

diameter = 26 cm

radius = diamtere/2

=26/2

=13 cm

area of circle  = πr^2

=3.14^13^2

=3.14*169

=530.66 cm^2

=531 cm^2   (after round off )

area of square= l^2

=5.1^2

=26.01 mi^2

are of rectangle = l *b

=6*5.1

=30.6 m^2

area of triangle = base*height / 2

=9*6.4 / 2

=57.6 / 2

=28.8 yd^2

Which number's estimate written as a single digit times a power of 10 will have a negative exponent?

Answers

105 i hope this helps if not then i’m sorry

Simplify:......................................................

Answers

Answer:

...

Step-by-step explanation:...

The cut off number is 2x-1

The probability distribution for a random variable x is given in the table X: -5,-3,-2,0,2,3 Probability: .17,.13,.33,.16,.11,.10 Find the probability that X <_-3

Answers

Answer:

0.3 probability that [tex]x \leq -3[/tex]

Step-by-step explanation:

The probability distribution is given in the table.

Probability that x <= -3

The values that are -3 or lower are -3 and -5. So

[tex]P(X \leq -3) = P(X = -3) + P(X = -5)[/tex]

From the table:

[tex]P(X = -3) = 0.13, P(X = -5) = 0.17[/tex]. So

[tex]P(X \leq -3) = P(X = -3) + P(X = -5) = 0.13 + 0.17 = 0.3[/tex]

0.3 probability that [tex]x \leq -3[/tex]

Answer:0.3

Step-by-step explanation:

What is the average score of runa 140,96 and 13?​

Answers

[tex]\huge\textsf{Hey there!}[/tex]

[tex]\large\textsf{Formula:}[/tex]

[tex]\mathsf{\dfrac{Sum\ of\ all\ terms}{Total\ number\ of\ values\ in\ the\ data}= your\ mean/average}[/tex]

[tex]\mathsf{\dfrac{140+96+13}{3}}[/tex]

[tex]\mathsf{140+36+13}[/tex]

[tex]\mathsf{140 + 96 = \bf 236}[/tex]

[tex]\mathsf{236 + 13}[/tex]

[tex]\mathsf{ = \bf 249}[/tex]

[tex]\mathsf{\dfrac{249}{3}}[/tex]

[tex]\mathsf{= \bf 83}[/tex]

[tex]\boxed{\boxed{\large\textsf{Possible answer: \huge \bf 83}}}\huge\checkmark[/tex]

[tex]\large\textsf{Good luck on your assignment and enjoy your your day!}[/tex]

~[tex]\frak{Amphitrite1040:)}[/tex]

How many liters each of a 25% acid solution and a 50% acid solution must be used to produce 80 liters of a 40% acid solution?

Answers

Answer:

32 and 48 liters

Step-by-step explanation:

Let 25% solution is x liters, then 50% solution is (80 - x) liters.

Acid content is going to be same:

0.25x + 0.5(80 - x) = 80*0.40.25x - 0.5x + 40 = 320.25x = 8x = 8/0.25x = 32 liters

So 32 liters of 25% solution and 80 - 32 = 48 liters of 50% solution

andrea is planning a birthday party and wants to include a cheese board with the desserts.
she reads online that she should have 110g of cheese per person ,but the cheese is sold in blocks of 500g
How many blocks of cheese should she buy to ensure that each guest can have 110g of cheese?​

Answers

Step-by-step explanation:

how many people in the party please ?

Calculate the area of the following Circle

Answers

Answer:

324

Step-by-step explanation:

Circles are 100% even. If you add another line going opposite direction you would multiply 18x18. You get 324.

he speeds (in MPH) of automobiles traveling in a city are given below:
20, 35, 42, 52, 65, 49, 24, 37, 23, 41, 50, 58
The mean speed of the cars is

Answers

Answer:

Mean speed = 41.3 mph

Step-by-step explanation:

Given that,

The speeds of an automobiles are given below:

20, 35, 42, 52, 65, 49, 24, 37, 23, 41, 50, 58

We need to find the mean speed of the cars.

Mean = sum of observations/ no. of observation

[tex]M=\dfrac{20+35+42+52+65+49+24+37+23+41+50+58}{12}\\\\M=41.3[/tex]

So, the mean speed of the cars is equal to 41.3 mph.

Given points (-3;-6), G(3; -2) and H(6; 1); determine:
(a) The equation of line FG

Answers

Answer:

The equation of line FG is [tex]y = \frac{2}{3}x - 4[/tex]

Step-by-step explanation:

Equation of a line:

The equation of a line has the following format:

[tex]y = mx + b[/tex]

In which m is the slope and b is the y-intercept.

F(-3;-6), G(3; -2)

When we have two points, the slope is given by the change in y divided by the change in x. So

Change in y : -2 - (-6) = -2 + 6 = 4

Change in x: 3 - (-3) = 3 + 3 = 6

Slope: [tex]m = \frac{4}{6} = \frac{2}{3}[/tex]

So

[tex]y = \frac{2}{3}x + b[/tex]

Finding b:

(3; -2) means that when [tex]x = 3, y = -2[/tex]. We use this to find b.

[tex]y = \frac{2}{3}x + b[/tex]

[tex]-2 = \frac{2}{3}(3) + b[/tex]

[tex]2 + b = -2[/tex]

[tex]b = -4[/tex]

The equation of line FG is [tex]y = \frac{2}{3}x - 4[/tex]

11x+7y=17
solve for y

Answers

[tex]\implies {\blue {\boxed {\boxed {\purple {\sf {\: y = \frac{17 - 11x}{7} }}}}}}[/tex]

[tex]\large\mathfrak{{\pmb{\underline{\red{Step-by-step\:explanation}}{\red{:}}}}}[/tex]

[tex]\\11x + 7y = 17[/tex]

[tex] \\➺ \: 7y = 17 - 11x[/tex]

[tex]\\➺ \: y = \frac{17 - 11x}{7} [/tex]

[tex]\bold{ \green{ \star{ \orange{Mystique35}}}}⋆[/tex]

The diagram shows a right-angled triangle.
xo
26 cm
17 cm
Find the size of angle x.
Give your answer correct to 1 decimal place.

Answers

Answer:

Diagram? I don't see a diagram.

Where is the diagram?

Step-by-step explanation:

solve marked question only !
plz

Answers

Answer:

hello ok I will share u the link where u can find step by step answers

Answer:

The angle of elevation of the top of a tower from a point on the ground, which is 30 m away from the foot of the tower, is 30°. Find the height of the tower.

A Let tower be AB

Let point be C

Distance of point C from foot of tower = 30m Hence,

BC = 30m

Angle of elevation = 30°

So < ACB = 30°

Since tower is vertical,

< ABC = 90°

I need help I’ll give u brainlest

Answers

Answer:

[tex]V=280[/tex] cubic inches

Step-by-step explanation:

Volume formula for triangular prism is  [tex]V=\frac{1}{2} bhl[/tex]

[tex]V=\frac{1}{2} (7)(10)(8)[/tex]

[tex]V=\frac{1}{2}(560)[/tex]

[tex]V=280[/tex]

Hope this helps

A geneticist conducts an experiment with peas, one sample of offspring consisted of 450 green peas and 157 yellow peas. Based on these results, estimate the probability of getting an offspring pea that is green.

Answers

Answer: 0.738

Step-by-step explanation:

11. Mendelian Genetics. When Mendel conducted his famous genetics experiments with peas, one sample of offspring consisted of 428 green peas and 152 yellow peas. Based on those results, estimate the probability of getting an offspring pea that is green. Is the result reasonably close to the value that was expected?

p0 = 428/(428 + 152) = 0.737931

If you round your answer of 0.737931 to three decimals you will

get 0.738.

What is the area of this figure?

Answers

Answer:

90km² only if it is parallelogram

Step-by-step explanation:

base = 9km

height=10km

area of parallelogram = b x h

=9km x 10km

=90km²

Answer:

A = 90km2

Step-by-step explanation:

Area of a rhombus is:

1. A = s x h (if given side and height)

2. A = 1/2 a x b (if given lengths of diagonals)

3. A = s^2 sin A (if given side and length)

Therefore from your problem, height and side is given thus, you'll use number 1

A = s x h

A = 9km x 10km = 90km2

Convex angles help me

Answers

Answer:

C, D, F

Step-by-step explanation:

Shape A is not a polygon; it has a line that doesn't connect anywhere. Even if it is a polygon, it would be concave. Shape B is a concave polygon, shape E is also a concave polygon, shape G and H are also concave polygons. Only shapes C, D, and F are convex polygons. Concave polygons are shapes that cave in, and convex polygons are caves that don't cave in.

Let V be the set of all 3x3 matrices with Real number entries, with the usual definitions of scalar multiplication and vector addition. Consider whether V is a vector space over C. Mark all true statements (there may be more than one).

a. The scalar closure axiom is satisfied
b. The additive inverse axiom is not satisfied
c The additive inverse axiom is satisfied
d. The additive closure axiom is not satisfied
e. The scalar closure axiom is not satisfied
f. The additive closure axiom is satsified
g. V is not a vector space over C
h. V is a vector space over C
i. The zero axiom is satisfied
j. The zero axiom is not satisfied

Answers

Answer:

the Scalar Closure axiom is not satisfiedV is not a Vector Space of CThe Additive Closure axiom is satisfied.

Step-by-step explanation:

According to the Question,

Given That, Let V be the set of all 3x3 matrices with Real number entries, with the usual definitions of scalar multiplication and vector addition. Consider whether V is a vector space over C.For V is a vector space over C and V is Set of 3x3 Matrices with Real entries.

Then, For any u,w ∈ V ⇒ u+w ∈ V

And u∈V and z∈C ⇒ z u ∈ V

So, If we take any z= i ∈ C

and V be any 3x3 matrices with Real entrices.

then, z,v ∉ V  ∴z,v Has Complex entries

So, the Scalar Closure axiom is not satisfied

Hence, V is not a Vector Space of C

Any u,w ∈ W ⇒ u+w ∈ V

So, The Additive Closure axiom is satisfied.

A person walks 1/6 mile in 1/18 hour.

The person's speed is _ miles per hour.

Answers

This Is What I Got!

Hope This Helps! :)

Have A Good Day!!

And If You Can I Wouldn't Mind A Brainliest! :))

Answer:

Divide 1/6 miles to 1/12hour since u wanna find our miles per hour

So it’ll be : 1/6 / 1/12

= 1/6 x 12/1

= 2 miles

Heyy!! Can someone help me please!!

3 (5x + 2) - 2 (4x -4)

I don’t know what to doooo!!

Answers

Answer:

7x + 14

Step-by-step explanation:

the first thing to do is expand the parentheses/brackets.

3(5x + 2) -2(4x - 4) will be

3(5x) + 3(2) -2(4x) -2(-4)

= 15x + 6 - 8x + 8

collect like terms

15x - 8x + 6 + 8 = 7x + 14

the answer is 7x + 14

Answer:

3(5x+2)-2(4x-4)

15x+6-8x+8

15x-8x+6+8

7x+14

Determine whether the following event is mutually exclusive or not mutually exclusive.

Choosing a student who is a mathematics major or a business major from a nearby university to participate in a research study. (Assume that each student only has one major.)

Answers

Answer:

The event is mutually exclusive.

Step-by-step explanation:

Mutually exclusive events are events that cannot exist simultaneously.

Thus, events that are not mutually exclusive can exist simultaneously.

Since each student only has one major, a single student cannot be both a mathematics major and a business major.

So, the event is mutually exclusive.

international system of 89643092 in words​

Answers

Answer:

Eighty nine million six hundred forty three thousand ninety two

Step-by-step explanation:

89,643,092

=> Eighty nine million six hundred forty three thousand ninety two

Please please help me please I really need help please just tap on picture and you will see the question

Answers

Answer:

No , it is not a right angle triangle

Step-by-step explanation:

according to the pythagoras theorem in right angled triangle sum of square of two sides is equal to the square of it's hypotenuse.

using pythagoras theorem

a^2 + b^2 = c^2

9^2 + 16^2 = 25^2

81 + 256 = 625

337 = 625

since sum of square of two smallest sides of a triangle is not equal to the square of it's hypotenuse it can be concluded that the given figure does not form right angle triangle.

Other Questions
please help :DDDDDDD If the amplitude of a sound wave is increased by a factor of four,how does the energy carry by the sound wave in each time interval change? The poem characterizes the oak tree as _______. How could you correctly combine the first three sentences of the passage below to add variety? Checksaw Amir. I was visiting New York. He lives there. We went to an Italian restaurant and ate big plates of spaghetti.was a fun outing.D saw Amir, who in lives in New York, when I was visiting.O When I was visiting New York, I saw Amir, who lives there.U When I was visiting New York; saw Amir, who lives there.0 saw Amir when I was visiting New York, which is where he lives. iSD I saw Amir, when I was visiting New York, that is where he livesANSWER IS A,B,D The numbers 1, 2, 3 , and 4 are drawn one at a time from the set {0, 1, 2, , 9}. If these four numbers are drawn with replacement, what is the probability that 14 23 is an even number? Which of the following best explains what may be happening in theother structures in the video based on what you know from thearticle. Help me pls this is hard The mean height of women in a country (ages 20-29) is 64 4 inches A random sample of 50 women in this age group is selected What is the probability that the mean height for the sample is greater than 65 inches? Assume o = 2.91 The probability that the mean height for the sample is greater than 65 inches is An electron is traveling with initial kinetic energy K in a uniform electric field. The electron comes to rest momentarily after traveling a distance d.a. What is the magnitude of the electric field? b. What is the direction of the electric field?1. in the direction of the electron's motion2. opposite to the direction of the electron's motion 3. perpendicular to the direction of the electron's motion Use______ to format cells when they meet a certain criteria.data calculationsconditional formattingdata formattingdata input Expand and simplify (n-2)(3n+7) + 2n(2n+3) PLZ HELPPP I need to pass this!! help will brainlist thx if a loaf of bread coast 1.52 how much change will you get from 10 if you buy 4loaves find the value of x. round to the nearest tenth. Which of the following is not generally a characteristic of metal?DuctilityOMalleabilityHigh melting pointLow boiling point Please, can I get help? The Marked price of a jacket at a dealer's shop is 2400. The dealer allows a discount of 15% to the retailer. As the company's policies the marked price of the retailer should be the same as the dealer's marked price. What SP should the retailer fix for the jacket? What would its final price be if 10% VAT is imposed? What would be the actual gain of retailer?Ans 2280,2502,240 Swifty Corporation has beginning work in process inventory of $128000 and total manufacturing costs of $277000. If cost of goods manufactured is $280000, what is the cost of the ending work in process inventory? a. $125000 b. $131000. c. $140000. d. $110000. Study the table about Earths interior.